You are on page 1of 15

03/05/2011

6.2. O critrio de Estabilidade de Routh-Hurwitz A. Hurwitz e E.J. Routh publicaram independentemente um mtodo de investigar a estabilidade de um sistema linear (vide Ogata). O critrio de estabilidade de Routh-Hurwitz verifica se todos os plos de uma funo de transferncia pertence ao semi-plano esquerdo do plano-s. Suponha que a funo de transferncia da forma:

b0s m + b1s m 1 + ... + bm 1s + bm G (s ) = , a0 0 a0s n + a1s n 1 + ... + an 1s + an


1passo: identifique apenas o denominador de G(s):

D (s ) = a0s n + a1s n 1 + ... + an 1s + an

(1)

2passo: verifique se qualquer destas constantes (ai) igual a zero ou, negativa na presena de pela menos uma constante positiva. Se isto ocorrer, conclua que o sistema instvel e no necessrio executar os prximos passos. Do contrrio, nada pode-se concluir, v para o 3 passo.
1

3passo: construa a seguinte tabela:

Os elementos a0, a1, ...,an so os coeficientes do denominador D(s) da equao (1).


2

03/05/2011

Os elementos b1, b2, b3, ..., c1, c2, ... e todos os demais so calculados com as seguintes expresses:

4passo: aplique o seguinte critrio de estabilidade de Routh-Hurwitz: O nmero de razes de D(s) (plos de G(s)) com parte real maior que zero (positivo) igual ao nmero de mudanas de sinal dos coeficientes da primeira coluna da tabela construda no 3passo. Obs.: se pelo menos um elemento da 1. coluna for nulo ou se uma linha toda for nula, deve-se observar o caso especial que mostraremos mais adiante. Exemplo: Seja

G (s ) =

2s + 1 s + 2s + 3s 2 + 4s + 5
4 3

estude sua estabilidade.

Sol.: 1 passo:

D (s ) = s 4 + 2s 3 + 3s 2 + 4s + 5

2passo: todos coeficientes de D(s) so positivos portanto nada pode-se concluir.

03/05/2011

3passo:

Neste caso, os elementos da 1coluna so:

Exemplo: Determine se o sistema estvel ou instvel:

Sol.:

1 passo:

s 2 + s 1 s 3 + 3s 2 s + 5 D (s ) = s 3 + 3s 2 s + 5 G (s ) =

2passo: existe um coeficiente negativo na presena de outro positivo, logo o sistema instvel e no precisa ir ao passo seguinte. Exerccio: O piloto automtico de um avio tem a seguinte F.T.M.F.:

(s ) 150s 3 + 900s 2 + 165s + 900 = 5 r (s ) s + 15s 4 + 240,5s 3 + 1303,6s 2 + 1667,4s + 924


verifique se o sistema estvel ou instvel.

>>den=[1 3 -1 5]; >>roots(den)

03/05/2011

O clculo dos plos de um sistema (razes de um polinmio) so fceis para os usurios do MATLAB ou das calculadoras cientficas atuais. Por exemplo, os plos do exemplo acima so calculados pelo MATLAB com o comando: >>den=[1 3 -1 5]; >>roots(den) Aparentemente o mtodo de Routh-Hurwitz seria desnecessrio, porm ele extramente til para projetar controladores, o exemplo a seguir ilustra este fato. Exemplo: Determine o intervalo de k, ganho do controlador, para o qual o sistema realimentado seja estvel.

sol.: A F.T.M.F. dada por:

(s + 1) k (s + 1) s (s 1)(s + 6) = H (s ) = (s + 1) s (s 1)(s + 6 ) + k (s + 1) 1+ k s (s 1)(s + 6)

Note que no possvel obter os plos de H(s) usando a calculadora. Usemos o mtodo de Routh-Hurwitz: 1passo: D(s)=s3+5s2+(k-6)s+k 2passo: Para que todos os coeficientes sejam positivos: k-6>0 k>6 e k>0 k>6 satisfaz (I)

03/05/2011

3passo:

Para que elementos da 1 coluna sejam todos positivos, necessrio que:

5( k 6) k 30 > 0 5k 30 k > 0 4k 30 > 0 k > k > 7,5 5 4


e k>0 (III)

(II)

Logo, para k>7,5 o sistema ser estvel. Como j foi dito, se tiver um zero (0) na primeira coluna de tabela ou se uma linha for nula, ento deve-se usar o caso especial a seguir.
9

Exerccio: Estude a estabilidade do sistema.

Exerccio: Estude a estabilidade do sistema.

10

03/05/2011

CASO ESPECIAL Se o primeiro elemento de uma linha zero, e pelo menos um elemento na mesma linha diferente de zero, ento substituiu-se o primeiro elemento de linha, que zero, por um pequeno nmero , que poder ser negativo ou positivo, e continua-se o clculo das prximas linhas da tabela. O exemplo abaixo ilustra este caso. Exemplo: Estude a estabilidade de

G (s ) =
Sol.:

5 s + 2s + 2s + 4s 2 + 11s + 10
5 4 3

1passo: D(s)=s5+2s4+2s3+4s2+11s+10 2passo: todos os coeficientes so positivos, nada pode-se concluir. 3passo: construo da tabela:

11

neste caso aparece um 0 na 1 coluna e outros elementos desta linha so diferentes de 0. Mostre que no possvel calcular os elementos da linha s2 pois seria necessrio dividir por 0. Substitua o 0 por e continue:

12

03/05/2011

para pequeno, 0, tem-se a seguinte tabela:

se 0 temos 6

13

Se 0 pela esquerda, ou seja <0, temos 2 trocas de sinais na primeira coluna. Se 0 pela direita, ou seja >0, temos tambm 2 trocas de sinais na primeira coluna. Assim, o sistema instvel.

14

03/05/2011

Exerccio: Estude a estabilidade de:

G (S ) =

7 s + 3s + 2s + 6s 2 + 6s + 9
5 4 3

Exerccio: Encontre a faixa de K tal que o sistema abaixo seja estvel:

Exerccio: Determinar a faixa de valores de K para a qual o sistema a seguir estvel:

15

Estabilidade de sistema com projeto de controlador dependente de dois parmetros Um controlador industrial muito utilizado o controlador P.I. (proporcional e integral). Neste caso a estabilidade fica dependente de dois parmetros. Um exemplo de projeto ilustra o uso do critrio de estabilidade de Routh-Hurwitz para este caso, e est mostrado a seguir. Exemplo: Para o sistema controlado por um controlador P.I. dado abaixo, encontre as faixas de kp e ki do controlador tal que o sistema abaixo seja estvel:

16

03/05/2011

Sol.: A F.T.M.F.

sk p + k i 1 . sk p + k i Y (s ) s (s + 1)(s + 2 ) = = (sk p + k i ) R (s ) s (s + 1)(s + 2) + sk p + k i 1 1+ . (s + 1)(s + 2) s

sk p + k i Y (s ) = 3 R (s ) s + 3s 2 + ( 2 + k p )s + k i
1passo: D(s)=s3+3s2+(2+kp)s+ki 2passo: para estabilidade necessrio que:

ki > 0 e: 2 + k p > 0 k p > 2

(I)

17

3passo: 1 coluna: 3( 2 + k

) ki > 0

kp >
e De (I), (II) e (III) tem-se a regio:

2 3 ki > 0

ki

(II) (III)

18

03/05/2011

Exerccio: Encontre a faixa de kp e ki do controlador abaixo tal que o sistema seja estvel.

Matlab (Simulink):

19

6.3. Estabilidade Relativa A estabilidade estudada at agora neste curso conhecida como estabilidade absoluta pois tem-se como referncia o lado esquerdo do planos. Um outro conceito o conceito de estabilidade relativa. Pode-se determinar a margem de segurana que um sistema apresenta no tocante sua estabilidade. Por exemplo, no plano-s abaixo, pode-se dizer que os plos z1 e z1 tem menor margem de estabilidade que os plos z2 e z3:

20

10

03/05/2011

Pode-se usar o critrio de Routh para estudar a margem de estabilidade relativa de um sistema, neste caso necessrio usar uma translao de eixo imaginrio. Os eixos acima so relacionados atravs da seguinte equao de translao de eixos:

ou ainda

s' = s + s = s '

21

Exemplo: Verifique se o sistema abaixo tem todos os plos esquerda de s=-1:

G (s ) =

1 s + 9s + 26s + 24
3 2

Sol.: Neste caso, deve-se realizar a translao de eixos abaixo:

logo, s=s-1 em G(s):

A translao do eixo imaginrio feita substituindo s=s-1 em G(s):

G (s ' ) =

1 (s '1) + 9(s '1)2 + 26(s '1) + 24


3
22

11

03/05/2011

ento,

G (s ' ) =

1 (s '1)(s ' 2s '+1) + 9(s ' 2 2s '+1) + 26s '26 + 24


2

G (s ' ) =
logo,

1 s ' +6s ' +11s '+6


3 2

este sistema estvel, sua estabilidade relativa engloba o eixo s=-1. Portanto sua margem de estabilidade >1. Obs.: Para determinar a margem de estabilidade (total) de um sistema necessrio ir transladando o eixo s (imaginrio) at o surgimento de um zero na 1 coluna da tabela de Routh-Hurwitz, indicando que existe plo sobre o eixo imaginrio s. 23

Este trabalho pode ser evitado, utilizando-se as calculadoras cientficas para obter todos os plos do sistema (ou o MATLAB), a margem de estabilidade ser igual ao mdulo da parte real do plo mais prximo ao eixo imaginrio, supondo-se que todos os plos so de sistema estvel. Exerccio: Use o MATLAB/SciLab ou a calculadora para determinar a margem de estabilidade do sistema.

G (s ) =

s s + 4s + 6s + 4
3 2

Exerccio: Verifique, usando o critrio de Routh-Hurwitz se o sistema abaixo tem todos seus plos esquerda de s=-2.

G (s ) =

s + 0,1 s 4 + 3s 3 + s 2 + 2s + 4

Exerccio: Projete K tal que o sistema abaixo tenha margem de estabilidade maior que 4.

24

12

03/05/2011

Exerccio: Use o MATLAB/SciLab ou a calculadora para determinar a margem de estabilidade do sistema. Soluo: No SciLab: -->s=poly(0,"s") s = s -->p=s^3+4*s^2+6*s+4 p = 2 3 4 + 6s + 4s + s -->roots(p) ans = - 1. + i - 1. - i - 2. -->s=poly(1,"s") s = -1+s

G (s ) =

s s + 4s + 6s + 4
3 2

-->p=s^3+4*s^2+6*s+4 p = 2 3 1+s+s+s -->roots(p) ans = - 1.145D-17 + i - 1.145D-17 - i - 1. -->s=poly(2,"s") s = -2+s -->p=s^3+4*s^2+6*s+4 p = 2 3 2s - 2s + s

-->roots(p) ans = 0 1. + i 1. - i -->n=s n = -2+s -->d=p d = 2 3 2s - 2s + s -->h=syslin('c',n./d); -->plzr(h)

25

26

13

03/05/2011

Exerccio: Verifique, usando o critrio de Routh-Hurwitz se o sistema abaixo tem todos seus plos esquerda de s=-2.

G (s ) =
Soluo:

s + 0,1 s + 3s + s 2 + 2s + 4
4 3

G (s ' ) =

(s '2 ) + 0,1 (s '2 ) + 3(s '2 )3 + (s '2 )2 + 2(s '1) + 4


4

G (s ' ) =

s '1,9 s ' 5s ' +7s ' 2 +2s '4


4 3

Usemos o mtodo de Routh-Hurwitz: 1passo:

D (s ' ) = s ' 4 5s '3 +7s ' 2 +2s '4

2passo: existe um coeficiente negativo na presena de outro positivo, logo o sistema instvel e no precisa ir ao passo seguinte.

27

No SciLab: -->s=poly(2,"s") s = -2+s -->n=s+0.1 n = - 1.9 + s -->d=s^4+3*s^3+s^2+2*s+4 d = 2 3 4 - 4 + 2s + 7s - 5s + s -->real(roots(d)) ans = 0.8124973 - 0.7009008 2.4442018 2.4442018 -->h=syslin('c',n./d); -->plzr(h)

28

14

03/05/2011

Exerccio: Projete K tal que o sistema abaixo tenha margem de estabilidade maior que 4.

Soluo: A F.T.M.F. dada por:

1 K 2 K s + 22s + 40 = s + 22s + 40 = H (s ) = 2 2 1 s + 22s + 40 + K s + 22s + 40 + K 1+ K . 2 s + 22s + 40 s 2 + 22s + 40

K.

Para que o sistema tenha margem de estabilidade maior que 4, s=-5, temos:

H (s ) =

K K = 2 (s '5) + 22(s '5) + 40 + K s ' +12s '45 + K


2

Usando o mtodo de Routh-Hurwitz: 1passo:

D (s ) = s ' 2 +12s '45 + K


29

2passo: para estabilidade necessrio que:

45 + K > 0 K > 45
3passo: As razes de D(s) so dadas abaixo, para K=46, pelo SciLab: No SciLab: -->s=poly(0,"s") s = s -->K=46 K = 46. -->p=s^2+12*s+45+K p = 2 1 + 12s + s -->roots(p) ans = - 0.0839202 - 11.91608 -->n=K n = 46. -->d=p d = 2 1 + 12s + s -->h=syslin('c',n./d); -->plzr(h)

30

15

You might also like